LSAT and Law School Admissions Forum

Get expert LSAT preparation and law school admissions advice from PowerScore Test Preparation.

User avatar
 Dave Killoran
PowerScore Staff
  • PowerScore Staff
  • Posts: 5852
  • Joined: Mar 25, 2011
|
#88141
Complete Question Explanation
(The complete setup for this game can be found here: lsat/viewtopic.php?f=167&p=88139#p88139)

The correct answer choice is (D).

If G is not on board when the van reaches S, then from the fourth rule J cannot be on board when the van reaches F. The third rule specifies that J is on board longer than V, and so we can also infer that V cannot be on board when the van reaches F. Answer choice (D) states that V is on board when the van reaches F, and so answer choice (D) cannot be true and is therefore correct.

The linkage between the third rule and the fourth rule creates the inference discussed above (and mentioned earlier). In every game, you must link rules through common variables and observe any consequences. The inference above is challenging, but it can be made during the setup of the game by using basic linkage.
 CEF
  • Posts: 9
  • Joined: Sep 03, 2013
|
#22128
Can you please explain question #23 on this?

Thank you!
 Nikki Siclunov
PowerScore Staff
  • PowerScore Staff
  • Posts: 1362
  • Joined: Aug 02, 2011
|
#22129
Hi CEF,

Question 23: if G is not on board when the van reaches Simcoe, it follows that J is not on board when the van reaches Fundy (this is the contrapositive of the first clause in the last rule). So, we have the following chain:

V > J > Fundy

So, V got off way before Fundy, proving answer choice (D) false.

Hope this helps!
 eober
  • Posts: 107
  • Joined: Jul 24, 2014
|
#16418
Hi,

How did we set up the template in this game. I had a very poorly constructed template for this game and I couldn't solve #23. Could you explain what the best way to diagram this game would be?

Thanks!
 David Boyle
PowerScore Staff
  • PowerScore Staff
  • Posts: 836
  • Joined: Jun 07, 2013
|
#16437
Hello,

One way to set it up:

la :arrow: 1,2 (or putting la/, /la over spaces 1, 2)
m :arrow: r still on board (though she could presumably get off at m)
v > j (v gets off before j)
f + j on board :dbl: s + g on board

For question 23,: from our final rule with the double arrow, then if g isn't on board at s, then j isn't on board at f. Since v has to get off before j (rule 3), then answer D, "v is on board at f", must be false.

Hope that helps,
David
 eober
  • Posts: 107
  • Joined: Jul 24, 2014
|
#16442
Makes perfect sense, thanks!
 xishao3
  • Posts: 6
  • Joined: Aug 18, 2017
|
#38978
Hi PowerScore,

I am still a bit confused on the set up of this problem. I made two inferences that slot 1 and 2 could only be filled by L or M when considering stops. Slots 3 and 4 could only be filled by F or S when considering stops. These two inferences were made keeping in mind the rule of J>V and the contrapositive of Rule 4 [G>S --> J>F]. I then proceeded to use the template approach keeping in mind these inferences. However, I still struggled to efficiently and smoothly come to the conclusion of Answer Choice D. May y'all help guide to a more efficient method?

Many Thanks,
Amy
 Adam Tyson
PowerScore Staff
  • PowerScore Staff
  • Posts: 5153
  • Joined: Apr 14, 2011
|
#39005
You may have made a bad inference or two here, Amy, starting with the placement of the stops. It may be that you are thinking either that "still on board when the van reaches X" means that the person doesn't get off at that stop. Consider the following order of stops as an example that shows your inferences to be incorrect:

FLMS

With F as the first stop, J is definitely still on board when it gets there. Everyone is! This setup would then tell us that G has to be the last one off the van, because he has to be on it at least until it gets to S, the final stop.

R would get off 3rd in this scenario, since she has to ride at least until the van gets to M, and she can't stay on it to S because that's G's stop.

Finally, since J is on longer than V, V must get off first, at F, and J gets off second, at L. The solution then looks like this:

VJRG
FLMS
1234

But wait, there's more! Could S be the first or second stop? Let's try it and see! How about an order of:

LSMF

Let's say G gets off first, and is therefore not on board when the van gets to S. That means H as to get off before the last stop, but after V, forcing him to get off 3rd, at M. V gets off before J, always, so V gets off at S, the 2nd stop, and R rides to the end of the line and gets off at F. She's cool, because she was still on the van when it got to M, the 3rd stop, and she kept on riding from there. That solution is:

GVJR
LSMF
1234

Looks like all the rules are satisfied, and the inference you made about L and M being the first two stops in either order is again proven incorrect.

Give some thought to why you made that inference, and see if it is related to a misunderstanding about one of the rules. "Still on board" doesn't mean they have to ride past it - they can get off at that stop! That does confuse more than a few students on this game, and may be at the root of your troubles here. If it helps, think about this being an airplane rather than a van. You're flying from LA to Atlanta with a layover in Phoenix. Are you still on the plane when it gets to Phoenix? I hope so! Can you get off there and say the heck with continuing to Altlanta? Sure you can!

Let us know if we can be of any further help. Keep at it!

Get the most out of your LSAT Prep Plus subscription.

Analyze and track your performance with our Testing and Analytics Package.